LSAT and Law School Admissions Forum

Get expert LSAT preparation and law school admissions advice from PowerScore Test Preparation.

 Administrator
PowerScore Staff
  • PowerScore Staff
  • Posts: 8917
  • Joined: Feb 02, 2011
|
#41253
Complete Question Explanation
(The complete setup for this game can be found here: lsat/viewtopic.php?t=6505)

The correct answer choice is (D)

An examination of the templates reveals that Q never works in the same zone as T, and thus answer choice (D) is correct.

While this question is an easy one with the templates, if you didn’t use that approach then keep in mind that prior work applies extremely well in problems of this nature. For example, the correct answer choice to question #12 shows Q and M working together, and this information eliminates answer choice (B). The solution scenario mentioned in question #15 (P, Q, S, and U working in Zone 1) can be used to eliminate answer choices (C) and (E). At that point, only two answers remain, and either you could abstractly solve the problem or simply create hypotheticals to determine the correct answer.

Get the most out of your LSAT Prep Plus subscription.

Analyze and track your performance with our Testing and Analytics Package.